is 25/4 grater then 40/8 ​

Answers

Answer 1

Answer:

yes it is greater:)

Answer 2

[tex]\huge\textsf{Hey there!}[/tex]

“[tex]\rm{Is \ \dfrac{25}{4} \ greater \ than \ \dfrac{40}{5}?}[/tex]”

[tex]\rm{In \order\ to \ find\ out \ which \ one \ is \ bigger, you \ can \ divide \ both \ fractions\ and\ you}\\\rm{can\ turn \ them \ into \ a \ decimal }[/tex]

[tex]\rm{\dfrac{25}{4}=\boxed{\rm{\bf 6.25}}}\large\checkmark[/tex]

[tex]\rm{\dfrac{40}{5}=\boxed{\rm{\bf 5}}}[/tex]

[tex]\rm{Usually, \ decimals\ and \ negative \ numbers \ are \ BELOW \ zero \ as\ positive \ numbers}\\\rm{and \ regular/real \ numbers\ are \ ABOVE \ zero }[/tex]

[tex]\boxed{\boxed{\large\textsf{So, therefore your ANSWER is: }\mathsf{\dfrac{25}{4}>\dfrac{40}{5}}}}\huge\checkmark[/tex]

[tex]\large\textsf{Good luck on your assignment and enjoy your day!}[/tex]

~[tex]\frak{Amphitrite1040:)}[/tex]


Related Questions

3 / 1/3 dividend by 1 / 1/5​

Answers

The answer for this question is 0.2

Answer two questions about Equations A and B:
A. 3x – 1= 7
B. Зx=8
1) How can we get Equation B from Equation A?

Answers

Answer:

1. c) Add/ Subtract the same quantity to/from both sides

2) Yes

Step-by-step explanation:

Answer two questions about Equations A and B:

A. 3x – 1= 7

B. Зx = 8

1) To get Equation B from Equation A we need to take the following steps

A. 3x – 1 = 7

We add one to both sides

3x - 1 + 1 = 7 + 1

3x - 0 = 8

3x = 8

Option c is the correct option.

2) We are given Equations:

A. 3x – 1= 7

B. Зx = 8

3x - 1 = 7 = 3x = 8

3x = 7 + 1 = 3x = 8

3x = 8 = 3x = 8

Therefore, we can say, Yes, Equation A is equivalent to Equation B, hence they have the same solution.

Question 3 of 10
In the diagram below, Ed is parallel to Xy. What is the value of x?

Answers

The answer is C because it interior angles and 180-105 =75

The value of y will be 75°. The correct option is C.

What are lines and angles?

Straight lines with little depth or width are present. You will learn about a number of lines, including transversal, intersecting, and perpendicular lines.

A figure called an angle is one in which two rays originate from the same point. In this area, you could also encounter contrasting and related viewpoints.

Given that the two parallel lines are DE and XY. The angle y will be calculated as,

y = 180 - 105

y = 75°

Hence, the measure of the angle y is 75°

To know more about lines and angles follow

https://brainly.com/question/28769265

#SPJ2

Which choice is equivalent to the expression? please help!

Answers

the answer is option e. *answer needs to be 20 characters long this should do :)

which of the two rational numbers 3/5 and -2/3 is greater

Answers

3/5 is greater than -2/3

Hi, does anyone know the answer to this question? I’m bad at geometry and I’m struggling to answer it.

Answers

The answer is (2) a translation

Please Mark BRAINLIEST!

A train traveling at a speed of 160 km / hr will take 18 hours to cover between two cities . How long will it take a car travelling at 120 km/hr to cover the same distance

Answers

Answer:

24 hours

Step-by-step explanation:

160 x 18 = 2880

2880/120 = 24

Hi, does anyone know the answer to this question? I’m bad at geometry and I’m struggling to answer it.

Answers

Answer:

QT = 16

Step-by-step explanation:

ΔQRS ~ ΔQRT

In similar triangles, corresponding angles are in same ratio.

[tex]\frac{QS}{QR}=\frac{QR}{QT}\\\\\frac{25}{20}=\frac{20}{QT}[/tex]

Cross multiply,

QT * 25 = 20 * 20

[tex]QT =\frac{20*20}{25}\\\\QT = 4*4\\\\QT = 16[/tex]

2) Simplify: 4/3+3/5+-3/5+-11/3​

Answers

Answer:

[tex]\frac{4}{3}+\frac{3}{5}+-\frac{3}{5}+-\frac{11}{3}[/tex][tex]=\frac{4}{3}-\frac{11}{3}[/tex][tex]\frac{a}{c}\pm \frac{b}{c}=\frac{a\pm \:b}{c}[/tex][tex]=\frac{4-11}{3}[/tex][tex]\:4-11=-7[/tex][tex]=\frac{-7}{3}[/tex][tex]\:fraction\:rule}:\quad \frac{-a}{b}=-\frac{a}{b}[/tex][tex]\frac{-a}{b}=-\frac{a}{b}[/tex][tex]\frac{-7}{3}[/tex][tex]=-\frac{7}{3}[/tex]---------------------[tex]\boxed{\boxed{\underline{\textsf{\textbf{hope it helps}}}}}[/tex][tex]\boxed{\boxed{\underline{\textsf{\textbf{have a great day!}}}}}[/tex]

A child weighs 44 lbs. The physician orders a medication for 10 mg/Kg/day. What do you give per day? (Use 0.45 for your conversion

Answers

Answer:

198 mg

Step-by-step explanation:

A child weighs 44 lbs.

Step 1

We convert the child's weight to kg

1 Ibs = 0.45kg

44 Ibs = x

Cross Multiply

1 Ibs × x = 44 Ibs × 0.45 kg

x = 44 Ibs × 0.45 kg/1 Ibs

x = 19.8 kg

Step 2

The physician orders a medication for 10 mg/Kg/day.

From the above statement, we know that:

10mg = 1 kg

Hence, what you give per day is calculated as:

1 kg = 10 mg

19.8kg = x

Cross Multiply

1 kg × x = 19.8kg × 10mg

x = 19.8kg × 10mg/1kg

x = 198mg

Therefore, what you will give per day to a child of 19.8kg(44 Ibs) is 198mg per day

2x-5y=-3
5x+2y=6
Is this a perpendicular lines ?

Answers

Perpendiculat Line is when both slopes of equation multiplying each others and equal to -1.

[tex] \large \boxed{m_1m_2 = - 1}[/tex]

For an easier way to understand, a perpendicular line has a negative reciprocal slope. For example if we are given the equation of y = 2x then the equation that is perpendicular to y = 2x would be y = (-1/2)x.

From both equations. We can either arrange in slope-intercepy form or use the slope formula which is m = -A/B when the equation is in Ax+By+C = 0 or Ax+By = C.

[tex] \large{ \begin{cases} 2x - 5y = - 3 \\ 5x + 2y = 6 \end{cases}}[/tex]

To find if these two lines are perpendicular to each others. Use the slope formula of -A/B.

[tex] \large{ \begin{cases} m_1 = - \frac{2}{ - 5} \\ m_2 = - \frac{5}{2} \end{cases}} \\ \large{ \begin{cases} m_1 = \frac{2}{ 5} \\ m_2 = - \frac{5}{2} \end{cases}}[/tex]

Now recall that it is perpendicular when one of them is negative reciprocal of one another (-5/2 is negative reciprocal of -(-2/5) = 2/5 and 2/5 is negative reciprocal of -5/2).

Or in definition of perpendicular lines, both slopes multiply and must equal to -1.

[tex] \large{( \frac{2}{5} )( - \frac{5}{2} ) = - 1} \\ \large{ - 1 = - 1}[/tex]

Thus the equation is true which makes both lines perpendicular to each others.

Answer

Both lines are perpendicular to each others.

Let me know if you have any doubts!

If cos theta = 0.8, find 1 / sin (pi/2 - theta)

Answers

Answer:

J

Step-by-step explanation:

Using the cofunction identity

cosθ = sin([tex]\frac{\pi }{2}[/tex] - θ )

Then

[tex]\frac{1}{sin(\frac{\pi }{2}-0) }[/tex]

= [tex]\frac{1}{cos0}[/tex]

= [tex]\frac{1}{0.8}[/tex]

= 1.25 → J

The value of 1/sin([tex]\frac{\pi }{2}[/tex] - θ) is 1.25.

What are four quadrants of trigonometry?

The coordinate axes divide the plane into four quadrants, labelled first, second, third and fourth as shown. Angles in the third quadrant, for example, lie between 180 degrees and 270 degrees.

Given

1/cosθ = 0.8

1/sin([tex]\frac{\pi }{2}[/tex] - θ) =?

By using quadrants in trigonometry

we know that sin([tex]\frac{\pi }{2}[/tex] - θ) = cosθ

= 1/sin([tex]\frac{\pi }{2}[/tex] - θ)

= 1/cosθ

= 1/0.8

= 1.25

1/sin([tex]\frac{\pi }{2}[/tex] - θ) = 1.25

Hence, the value of 1/sin([tex]\frac{\pi }{2}[/tex] - θ) is 1.25.

Learn more about four quadrants of trigonometry here

https://brainly.com/question/21864197

#SPJ2

Marc is building a rectangular wooden frame for his canvas. If the canvas is 7 feet long by 5 feet wide, what is the perimeter of the canvas?

Answers

Answer:

24 feet

Step-by-step explanation:

Use the perimeter formula, P = 2l + 2w, where l is the length and w is the width.

Plug in the values, and solve:

P = 2l + 2w

P = 2(7) + 2(5)

P = 14 + 10

P = 24

So, the perimeter of the canvas is 24 feet

A mistake was made in the steps shown to simplify the expression. Which step includes the mistake?
1 + 92
5
+ -10% = 2
Step 1: = 1 + 32 + 10 = 2
Step 2:=
1* ° + 10 = 2
1 + 10 = 2
Step 4: = 2 + 10 = 2
Step 5: = 12 = 2
Step 6: = 6
Step 3:=
O A. Step 4
doorn
O B. Step 5
O C. Step 1
D.
Step 3
M

Answers

Answer:

step 4

Step-by-step explanation:

Answer:

Step  4

Step-by-step explanation:

[tex]\frac{1 + 3^2}{5} + |-10| \div 2\\\\Step \ 1 : \frac{1 + 3^2}{5} + 10 \div 2\\\\step \ 2 : \frac{1 + 9}{5} + 10 \div 2 \\\\Step \ 3 : \frac{10}{5} + 10 \div 2\\\\Step\ 4: 2 + 10 \div 2\\\\Step \ 5 : 2 + 5 \ \ \ \ \ \ \ \ \ \ \ \ \ [ \ given : 12 \div 2 \ ]\\\\Step \ 6 : 7[/tex]

Homework June 10th 2021 Jim

Answers

Problem 1

Part (a)

It is possible to find the equation of the line of best fit by hand, but it's much more efficient to use technology. That could mean a graphing calculator, an online tool, or some computer software installed. I'm going to use GeoGebra. Specifically, I'm using the "FitLine" command to find the regression line.

The equation of line of best fit is y = -2.45x + 11.83

There's not much to say in terms of steps, since it's basically a calculator problem. Doing this by hand would take a lot longer than it should be.

------------

Part (b)

Plug in x = 2.3 and evaluate

y = -2.45x + 11.83

y = -2.45*2.3 + 11.83

y = 6.195

Answer: 6.195

-------------

Part (c)

We'll compare the result from the previous part (6.195) to the y value in the table (6.2)

The difference is 6.2 - 6.195 = 0.005

This means the estimated value is off by 0.005 and this is an underestimate (since 6.195 is smaller than 6.2)

===================================================

Problem 2

To be perfectly honest, I'm not sure what's going on here. It seems like there's missing context to the problem. Perhaps a data table that got cut off or this is referring to a previous problem.

===================================================

Problem 3

Recall that y = mx+b is the slope intercept form

m = slope

b = y intercept

We simply read off the values of the given equation y = 0.404x - 5.18 to see that 0.404 is the slope and -5.18 is the y intercept

The slope tells us that each time x goes up by 1, y will increase by an estimated amount of 0.404; this represents the unit rate or speed. In this context, it means the height goes up by about 0.404 cm per day. This is an estimated value because the regression line itself is a collection of estimated points.

The y intercept is where the graph crosses the y axis. It always occurs when x = 0. The x refers to the day number. Day 0 is basically the starting day. So the y intercept being -5.18 means the estimated height is -5.18 cm on the starting day. At first glance, it might not seem possible to have a negative height. But simply think of negative heights as below ground level, much like how the negative y values are below the x axis.

In other words, the plant is estimated to start off at about 5.18 cm below ground level. This is a reasonable assumption because the seed is buried into the ground (assuming at this level more or less) and it then grows upward.

if you know the answer please help me ese ​

Answers

Answer:

Domain { 4,8,12,16,20}

Range { 1,2,3,4}

This is not a function because two of the inputs have more than one output

Step-by-step explanation:

The domain is the x values ( input)

Domain { 4,8,12,16,20}

The range is the y values (output)

Range { 1,2,3,4}

This is not a function because two of the inputs have more than one output

Someone help please struggling on this answer thank you so much!

Answers

Answer:

-x+8=6

Step-by-step explanation:

x+5-1(2x)+(-1)(-3)=6

x+5-1-2x+3=6

ans (-x+8=6)

x+8

I have $5$ different mathematics textbooks and $4$ different psychology textbooks. In how many ways can I place the $9$ textbooks on a bookshelf, in a row, if there must be a psychology textbook exactly in the middle, and there must be a mathematics textbook at each end?

Answers

Answer:

What

Step-by-step explanation:

You can’t have more than 2 combinations with £9

What is the difference between the ordinates of the points ( -2 , 3 ) and ( 5, 8 ) ?

Answers

Answer:

the difference in the coordinate points is (7, 5)

Step-by-step explanation:

Given;

first coordinate point, (x₁, y₁) = (-2, 3)

second coordinate point, (x₂, y₂) = (5, 8)

The difference in the coordinate points is calculated as follows;

Δ(x, y) = (x₂ - x₁,  y₂ - y₁)

          = (5 - -2,  8 - 3)

         = (7, 5)

Therefore, the difference in the coordinate points is (7, 5)

A circle has a radius of 18 ft and an arc length of 117. What is the measure of the central angle of
the circle?

Answers

20akakdjhajaksakakka

44
Match each ratio with its simplest form.
12:15
20:36
18:24

Pls answer ASAP I need to finish this today and I’m stuck

Answers

12 : 15 = 4 : 5

20 : 36 = 5 : 9

18 : 24 = 3 : 4

Which is equivalent to
1/4x

Answers

Answer:

1/4x is basically x divided by 4/multiplied by 1/4

this can be applied in examples such as:

1/4(2) = 0.5

etc.

Step-by-step explanation:

i hope this helped, if not, please leave a comment specifically requiring what exactly you are stuck with.

Answer:

 1/8x+1/8x

Step-by-step explanation:

please help me immediately​

Answers

Answer:

zero of a polynomial can be defined as the points where the polynomial become 0 as a whole.

An equation formed with variables, exponents and coefficient together with operation and an equal sign is called polynomial equation.

I hope this will help you

Jaclyn used the slope formula to find the slope of the line through the points given in the table.

A 2-column table with 2 rows. Column 1 is labeled x with entries 0, negative 3. Column 2 is labeled y with entries 4, 0.

1. Point 1: (0, 4); Point 2: (negative 3, 0). 2. Slope = StartFraction y 2 minus y 1 Over x 2 minus x 1 EndFraction = StartFraction 0 minus 4 Over negative 3 minus 0 EndFraction = StartFraction negative 4 Over negative 3 EndFraction. 3. Slope = four-thirds.

In which step did Jaclyn make an error?

Answers

Answer: She didn't make an error.

Step-by-step explanation:

She just didn't I got it right.

Answer:

answer is D

Step-by-step explanation:

factorise the following (a+b)⁴-(a-b)⁴​

Answers

Answer:

hope it is helpful to you

this is another answer

sorry for bad handwriting

actually its my sista handwriting

3/4x + 12 =48 what is the value of ' x '​

Answers

Answer:

3/4x + 12 =48

3/4x=36

x=48

can some one pls help me with this i need help

Answers

Answer:

9/8 cup peanuts were used

The amounts are:

Correct

Correct

Incorrect

What is the best name for the figure with vertices at the following coordinates?
(0,0), (-1, 1), (6, 1), and (2, 0)
A.
rectangle
B.
square
C.
trapezoid
D.
rhombus

Answers

Answer:  C) Trapezoid

Refer to the figure below. Note how sides BC and AD are parallel. We have exactly one pair of parallel sides, so that's why we have a trapezoid.

The figure with the vertices (0,0), (-1, 1), (6, 1), and (2, 0) is a trapezoid

What is a trapezoid?

The Trapezoid is a 4 sided polygon. Two sides of the shape are parallel to each other and they are termed as the bases of the trapezoid. The non-parallel sides are known as the legs or lateral sides of a trapezoid.

There are three types of trapezoids , and those are given below:

a) Isosceles Trapezoid

b) Scalene Trapezoid

c) Right Trapezoid

The area of the Trapezoid is given by

Area of Trapezoid = ( ( a + b ) h ) / 2

where , a = shorter base of trapezium

b = longer base of trapezium

h = height of trapezium

Given data ,

Let the coordinates of the figure be represented as ABCD

Now , the vertices of the figure is

A ( 0 , 0 ) , B ( -1 , 1 ) , C ( 6 , 1 ) and D ( 2 , 0 )

On plotting the coordinates in the graph , we get a trapezoid with

The longer base = BC

The shorter base = AD

So , the figure is a trapezoid

Hence , the vertices represents a trapezoid

To learn more about trapezoid click :

https://brainly.com/question/12221769

#SPJ7

Determine the domain of the following graph:

Answers

answer:

(-1,8]

explanation I just did something close to this

The tennis courts are 3 blocks west and 2 blocks south of Miguel’s house. The bike path is 5 blocks east and 2 blocks south of Miguel’s house. How many blocks are the tennis courts from the bike path?

Answers

Answer:

8

Step-by-step explanation:

If Miguel's house is (0,0), then the tennis court is (-3,-2), and the bike path is (5,-2). There fore, we can just count the squares between the tennis court and the bike path, which is 8.

Hope that this helps!

From the coordinate calculation, here tennis courts are 8 blocks far from the bike path.

Given that,
The tennis courts are 3 blocks west and 2 blocks south of Miguel’s house. The bike path is 5 blocks east and 2 blocks south of Miguel’s house. How many blocks are the tennis courts from the bike path is to be detemined.


What is coordinate?

Coordinate, is represented as the values on the x-axis and the y-axis of the graph.

Here on the graph,
Assume that Miguel's house is (0,0),
then accoding to the question the tennis court is (-3,-2), and the bike path is (5,-2).
Now, difference in the position dx = 5 - (-3)
                                                    = 8

Thus, from the coordinate calculation, here tennis courts are 8 blocks far from the bike path.


Learn more about coordinate here:

brainly.com/question/13498438

#SPJ2

Other Questions
Which inequality has -12 in its solution set?ABDX+6 Hey help me please much help ( Just answer 1 & 2) Please help! This is the last thing I have to do. I will mark brainliest asap!! What is the purpose of the essay? A. to show why acts of road rage should be legal B. to show why people need to be overly defensive C. to convince crazy drivers to become safe drivers A model train is built at a scale of 1 to 60. If the model train is 10 inches long, how many feet long is the actual train? A group of oceanographers measured the water temperature of the Atlantic Ocean at various depths and found that the temperature is different. Which of the objects are the independent variable and which is the dependent variable? Given that x/1 = 3y/8, find the ratio of : y 6/8 - h/8 = 1/8What is the value of h?Please answer, I really need help. who controlled the economy in ancient mesopotamia Why did the US demilitarize Japan? A customer borrows $450 at 5% interest compounded annually 1. Prepare the December 31 adjusting entries for the following transactions. Omit explanations. 1. Fees accrued but not billed, $6,300. 2. The supplies account balance on December 31, $4,750; supplies on hand, $960. 3. Wages accrued but not paid, $2,700. 4. Depreciation of office equipment, $1,650. 5. Rent expired during year, $10,800. A battery-powered set of five patio lanterns is connected in series. An ammeter measures the current through the battery as 0.75 A. The total resistance of the circuit is 52 0. (a) Calculate the voltage drop across the battery. (b) Calculate the voltage drop across each load. piano research paper 3n - 5 = -8what is the value of n? Answer questions #13 & 14. Number 13: What is the system shown? Number 14: Type the correct answer in the box. THE IMAGE BELOW SHOWS BOTH QUESTIONS. PLZ HELP LAST QUESTION!!! MESSENGER please help fast!!!!!!!!! use quadratic formula to solve these equation (a).3x+6x+2=0 what you inspire you about the story of IAS officer Mr Govinda 150 to 200 words